Question:

Observe the following equilibrium $$\text{Fe}^{3+}(aq) + \text{SCN}^-(aq) \rightleftharpoons [\text{Fe(SCN)}]^{2+}(aq)$$ yellow \hspace{1cm colourless \hspace{1cm} deep red}
Addition of aqueous oxalic acid solution to the above equilibrium

Show Hint

Le Chatelier's principle is crucial for predicting the direction of equilibrium shifts when conditions like concentration, temperature, or pressure are changed. Remember that removing a reactant or product will shift the equilibrium towards the side where that species is present.
Updated On: May 13, 2025
  • Shifts the equilibrium towards the formation of $[\text{Fe(SCN)}]^{2+}$
  • Deep red color increases
  • Intensity of deep red color decreases
  • No change in equilibrium
Hide Solution
collegedunia
Verified By Collegedunia

The Correct Option is C

Solution and Explanation

Step 1: Understand the reaction of oxalic acid with Fe$^{3+$ ions.}
Oxalic acid (H$_2$C$_2$O$_4$) reacts with Fe$^{3+}$ ions to form stable complex ions:
$$\text{Fe}^{3+}(aq) + \text{C}_2\text{O}_4^{2-}(aq) \rightleftharpoons [\text{Fe(C}_2\text{O}_4)]^{1+}(aq)$$ This reaction removes Fe$^{3+}$ ions from the equilibrium system.
Step 2: Apply Le Chatelier's principle.
The given equilibrium is:
$$\text{Fe}^{3+}(aq) + \text{SCN}^-(aq) \rightleftharpoons [\text{Fe(SCN)}]^{2+}(aq)$$
The removal of a reactant (Fe$^{3+}$) will shift the equilibrium to the left to counteract this change and restore equilibrium.

Step 3: Analyze the effect on the concentration of $[\text{Fe(SCN)}]^{2+$.}
The shift to the left means the reverse reaction is favored, consuming $[\text{Fe(SCN)}]^{2+}$ ions and decreasing their concentration in the solution.
Step 4: Relate the concentration of $[\text{Fe(SCN)}]^{2+$ to the color intensity.}
The deep red color of the solution is due to the presence of $[\text{Fe(SCN)}]^{2+}$ ions. A decrease in the concentration of these ions will lead to a decrease in the intensity of the deep red color. Final Answer: The final answer is $\boxed{Intensity of deep red color decreases}$
Was this answer helpful?
0
0